google is your friend

来源: jackyhan 2012-07-12 09:11:11 [] [旧帖] [给我悄悄话] 本文已被阅读: 次 (1513 bytes)
回答: 求解一道数学题孺将2012-07-12 08:44:19

The answer is: n=3

Here are two typical solutions:



There in all 3n(3n-1)/2 matches.  These can be broken down into M-M,
M-F, an d F-F.  The number of M-M matches is n(2n-1).  Since all of
these must be won by men,

n(2n-1) <= (5/12) (9n^2 -3n)/2, which solves to  n<= 3.

We are then left with three possible scenarios:
6 men; 3 women
4 men; 2 women
2 men; 1 woman

One can begin with any of these, but it can be found that the first
works in the case that women beat men in all mixed matches.  The
others do not have enough matches for there to be a 7/5 ratio (i.e.
the number of matches is not divisible by 12).




The total number of matches is 3n(3n-1)/2.  Of these, 5n(3n-1)/8 (5/12
of the total) were won by men.  Since this number must be an integer,
n = 0 or 3 (mod 8).

There were n(2n-1) matches between men, and of course all of these
matches were won by men.  Therefore, the number of mixed matches won
by men is 5n(3n-1)/8 - n(2n-1) = (-n^2 + 3n)/8.

The total number of mixed matches is 2n^2, so 0 <= (-n^2 + 3n)/8 <=
2n^2.  The right side of the inequality will hold for any n >= 1, and
the left side will hold for n <= 3.

CONCLUSION: There were 3 women and 6 men.  All of the mixed matches
were won by women.


请您先登陆,再发跟帖!

发现Adblock插件

如要继续浏览
请支持本站 请务必在本站关闭/移除任何Adblock

关闭Adblock后 请点击

请参考如何关闭Adblock/Adblock plus

安装Adblock plus用户请点击浏览器图标
选择“Disable on www.wenxuecity.com”

安装Adblock用户请点击图标
选择“don't run on pages on this domain”